MGIMS WARDHA MGIMS WARDHA Solved Paper-2004

  • question_answer
    Figure shows three points A, B and C in a region of uniform electric field \[\mathbf{\vec{E}}.\]The line AB is perpendicular and BC is parallel to the field lines. Then which of the following holds good?

    A) \[{{V}_{A}}={{V}_{B}}={{V}_{C}}\]                           

    B) \[{{V}_{A}}={{V}_{B}}>{{V}_{C}}\]

    C) \[{{V}_{A}}={{V}_{B}}<{{V}_{C}}\]                           

    D) \[{{V}_{A}}={{V}_{B}}>{{V}_{C}}\]

    Correct Answer: B

    Solution :

    Where \[{{V}_{A}}>{{V}_{B}}\] and \[{{V}_{C}}\] represent the electric potential at points A, B and C respectively. Electric lines of force flow from higher potential to lower potential so, \[{{V}_{A}}={{V}_{B}}>{{V}_{C}}\]


You need to login to perform this action.
You will be redirected in 3 sec spinner